Do you know anything about y=mx+b​

Answers

Answer 1

Answer:

yes

Step-by-step explanation:

your m is your slope,your b is the y-intercept, and your y is what you are solving for.

Answer 2

Answer:

Y=mx+b is an equation for a linear equation

M is the slope of the line and B is the y-intercept

The y-intercept of this line is the value of y at the point where the line crosses the y axis.

Every straight line can be represented by an equation: y = mx + b.

Hope this helps :))


Related Questions

(1 point) find the area of the region outside r=5 5sinθ , but inside r=15sinθ.

Answers

The area of the region outside r=5sinθ and inside r=15sinθ is 50π square units.

The area of the region outside r=5sinθ and inside r=15sinθ, we need to evaluate the integral of the area element dA over the region of interest. The area element in polar coordinates is given by dA = r dr dθ.

The region of interest is the annular region between the two circles, which is defined by the inequalities:

5sinθ ≤ r ≤ 15sinθ

0 ≤ θ ≤ π

Thus, the area of the region is given by:

A = [tex]\int\int dA = \int_0^\pi \int_5sin\theta^{(15sin\theta)} r dr d\theta[/tex]

Using the limits of integration, we can rewrite the integral as:

A = [tex]\int_0^\pi [1/2 (15sin\theta)^2 - 1/2 (5sin\theta)^2] d\theta[/tex]

Simplifying the integrand, we get:

A = [tex]1/2 \int_0^\pi (225sin^2\theta - 25sin^2\theta) d\theta[/tex]

A = [tex]1/2 \int_0^\pi 200sin^2\theta d\theta[/tex]

Using the identity sin²θ = 1/2 - 1/2cos2θ, we get:

A =[tex]1/2 \int_0^\pi 100 - 100cos2\theta d\theta[/tex]

Integrating, we get:

A = 1/2 [100θ - 50sin2θ] from 0 to π

A = 1/2 [100π - 0] - 1/2 [0 - 0]

A = 50π

For similar questions on area

https://brainly.com/question/25292087

#SPJ11

The area of the region outside the polar curve r = 5sinθ and inside the polar curve r = 15sinθ is 50π square units.

To calculate the area between two polar curves, we integrate the outer curve and subtract the integral of the inner curve over the desired interval. In this case, the curves are r = 5sinθ and r = 15sinθ, and we want to find the area from θ = 0 to θ = π.

The equation r = 5sinθ represents the inner curve, and r = 15sinθ represents the outer curve.

Using the formula for the area between two polar curves, the area A can be calculated as follows:

A = (1/2) ∫[θ1,θ2] (r_outer^2 - r_inner^2) dθ

Substituting the given equations, we have:

A = (1/2) ∫[0,π] ((15sinθ)^2 - (5sinθ)^2) dθ

Simplifying the equation further:

A = (1/2) ∫[0,π] (225sin^2θ - 25sin^2θ) dθ

A = (1/2) ∫[0,π] 200sin^2θ dθ

Integrating this equation over the given interval, we get:

A = (1/2) * 200 * ∫[0,π] sin^2θ dθ

Using the identity ∫ sin^2θ dθ = (1/2) * (θ - sinθcosθ), we have:

A = (1/2) * 200 * [(π - sinπcosπ) - (0 - sin0cos0)]

A = (1/2) * 200 * [(π - 0) - (0 - 0)]

A = (1/2) * 200 * π

A = 100π

Finally, we subtract the area enclosed by the inner curve r = 5sinθ to get the area between the curves:

A = 100π - (1/2) * 5^2 * π

A = 100π - 25π

A = 75π

Therefore, the area of the region outside r = 5sinθ but inside r = 15sinθ is 50π square units.

To learn more about polar curves click here

brainly.com/question/27548409

#SPJ11

A popular podcast wants to know the proportion of listeners that think assault weapons should be banned for civilians. Listeners are asked to text "Y" for yes or "N" for no to a provided number. Sixty-five percent of the 1,500 people that responded texted "Y." Which condition for inference has NOT been met?A) All conditions appear to be met.B) The sample is an SRS of the population.C) N > 10nD) np ≥ 10 and n(1 - p) ≥ 10E) Inference about a proportion is the objective.

Answers

Based on the information provided, it appears that all conditions for inference have been met. The correct option is option (A).

The sample size is large enough (n=1500) to meet the condition of np ≥ 10 and n(1 - p) ≥ 10.

The sample is also random (as listeners are asked to text in) and independent, so option B is met.

There is no indication that the sample is less than 10% of the population, so option C is met.

Finally, the objective of the inference is to estimate the proportion of listeners who think assault weapons should be banned for civilians, so option E is also met.

Therefore, all conditions appear to be met and no condition for inference has not been met.

To know more about inference refer here :

https://brainly.com/question/30267509#

#SPJ11

Use Greens Theorem to find the counterclockwise circulation and outward flux for the field F = (6y2 ? x2)i - (x2 +6y2)j and curve C: the triangle bounded by y = 0, x= 3, and y = x. The flux is . (Simplify yow answer) The circulation is . (Simplify your answer)

Answers

The counterclockwise circulation of F is 99

The flux F across C is -99

Define the area of integration

C: Triangle bounded by

x = 0, y = 0 , y = x

[tex]0\leq x\leq 3,0\leq y\leq x[/tex]

Applying Green's Theorem for counterclockwise circulation

[tex]F=y^2-6x^2i+6x^2+y^2j[/tex]

[tex]I=\int\limits_C P(x,y)dx+Q(x,y)dy=\int\limits\int\limits_D(\frac{dQ}{dx}-\frac{dP}{dy} )dA[/tex]

[tex]p(x,y)=y^2-6x^2---- > \frac{dP}{dy}=2y\\ \\Q(x,y)=6x^2+y^2---- > \frac{dQ}{dx}=12x\\ \\I=\int\limits\int\limits_D 12x -2y dA[/tex]

Calculate the integral. (With respect to the x axis)

[tex]I=\int\limits^3_0 \int\limits^x_0 {12x}-2y \, dydx\\ \\I=\int\limits^3_0 {12x}-y^2|^x_0 \, dx \\\\I=\int\limits^3_0 11x^2\, dx\\\\I=\frac{11x^3}{3}|^3_0\\ \\I=99[/tex]

Applying Green's Theorem for flux of the field

[tex]F=y^2-6x^2i+6x^2+y^2j[/tex]

[tex]\int\limits\int\limits_D(\frac{dQ}{dx}+\frac{dP}{dy} )dA[/tex]   the flux across the C

[tex]p(x,y)=y^2-6x^2---- > \frac{dP}{dx}=-12x\\ \\Q(x,y)=6x^2+y^2---- > \frac{dQ}{dy}=2y\\ \\I=\int\limits\int\limits_D 2y-12x dA[/tex]

Calculate the integral. (With respect to the x axis)

[tex]I=\int\limits^3_0 \int\limits^x_0 {2y}-12x \, dydx\\ \\I=\int\limits^3_0 y^2-12xy|^x_0 \, dx \\\\I=\int\limits^3_0- 11x^2\, dx\\\\I=-\frac{11x^3}{3}|^3_0\\ \\I=-99[/tex]

The counterclockwise circulation of F is 99

The flux F across C is -99

Learn more about Greens Theorem at:

https://brainly.com/question/30763441

#SPJ4

The given question is incomplete, So i take similar question:

Use Green's theorem to find the counterclockwise circulation and outward flux for the field[tex]F=(y^2 - 6x^2) i + (6x^2 + y^2) j[/tex]  and curve C: the triangle bounded by y=0, x=3 and y=x. What is the flux and circulation?

statstics paroxysmal nocturnal hemoglobinuria is an extremely rare, acquired life threatening disease of the blood. in pnh the bone marrow produces. suppose that the probability

Answers

Paroxysmal nocturnal hemoglobinuria (PNH) is a rare blood disorder that can be life-threatening.

The condition occurs when the bone marrow produces abnormal red blood cells that are destroyed by the immune system.

This destruction of red blood cells can lead to a wide range of symptoms, including fatigue, shortness of breath, abdominal pain, and blood clots.
When it comes to statistics, it is important to note that PNH is an extremely rare disease.

According to the National Organization for Rare Disorders (NORD), the incidence of PNH is estimated to be between 1 and 5 cases per million people.

This means that the probability of developing PNH is very low.
The exact cause of PNH is not fully understood, but it is thought to be an acquired genetic mutation that affects the way red blood cells develop.

There are currently no known cures for PNH, but there are treatments available that can help manage the symptoms and improve quality of life.
In conclusion, while PNH is a serious and rare disease, the probability of developing it is very low.

It is important for individuals who are experiencing symptoms of PNH to speak with their healthcare provider to receive a proper diagnosis and discuss treatment options.

Know more about blood clots here:

https://brainly.com/question/150122

#SPJ11

Let f(t) be the temperature (in degrees Celsius) of a liquid at time t (in hours). The rate of temperature change at time a has the value f(a). Determine the proper method of solution for the question.By how many degrees did the temperature rise during the first 4 hours?Which of the following will result in the number of degrees the temperature of the liquid rose during the first 4 hours?OA Compute f'(4).OB. Compute 1(4).OC. Subtract the liquid's initial temperature from its temperature 4 hours later.OD. Subtract the liquid's initial temperature from its temperature 4 hours later and divide by 4.

Answers

The proper method of solution for the question "By how many degrees did the temperature rise during the first 4 hours?" is to subtract the liquid's initial temperature from its temperature 4 hours later, which is option (C).

To find the change in temperature, we need to calculate the temperature difference between the initial and final temperatures of the liquid. Since we are asked about the temperature rise, we need to subtract the initial temperature from the temperature after 4 hours. This gives us the total increase in temperature. Option (A) is incorrect because it only gives the value of the rate of change of temperature at time 4, but not the temperature change over the entire 4 hour period. Option (B) is also incorrect, as it does not provide any information about the temperature at all. Option (D) is incorrect because dividing by 4 assumes that the temperature change is constant over the entire 4 hour period, which may not be true. Therefore, option (C) is the correct method of solution to find the number of degrees the temperature of the liquid rose during the first 4 hours.

Learn more about temperature here

https://brainly.com/question/26866637

#SPJ11

Solve for 18 points!!

Answers

Answer: 9

explanation: 6x4 is 24 - 15 = 9

Answer:

b = 9

Step-by-step explanation:

Solve: [tex]\frac{b+15}{6}[/tex] = 4

[tex]\frac{b+15}{6}[/tex] = 4

b + 15 = 24

b = 24 - 15

b = 9

Given a standard normal distribution, find the value of k such that (a) P(Z > k) = 0.2046: (b) P(Z < k) = 0.0427: (c) P(-0.93 < Z < k) = 0.7235.

Answers

The value of k for part (c) is 0.15.

(a) To find the value of k such that P(Z > k) = 0.2046, we need to look up the z-score that corresponds to a cumulative probability of 1 - 0.2046 = 0.7954. Using a standard normal table or a calculator, we can find that the z-score for this probability is approximately 0.84. Therefore, k = -0.84.

(b) Similarly, to find the value of k such that P(Z < k) = 0.0427, we need to look up the z-score that corresponds to a cumulative probability of 0.0427. Using a standard normal table or a calculator, we can find that the z-score for this probability is approximately -1.71. Therefore, k = -1.71.

(c) To find the value of k such that P(-0.93 < Z < k) = 0.7235, we need to first find the z-score that corresponds to a cumulative probability of (1 - 0.7235)/2 = 0.13825, which is the probability to the left of -0.93. Using a standard normal table or a calculator, we can find that the z-score for this probability is approximately -1.08.

Then, we need to find the z-score that corresponds to a cumulative probability of 1 - 0.13825 = 0.86175, which is the probability to the right of k. Using a standard normal table or a calculator, we can find that the z-score for this probability is approximately 1.08.

The value of k can be found by adding the z-scores for the probabilities to the left and right of k: k = -0.93 + 1.08 = 0.15. Hence, the value of k for part (c) is 0.15.

To know more about probability refer to-

https://brainly.com/question/30034780

#SPJ11

A culture of bacteria in a particular dish has an initial population of 400 cells grows at a rate of N'(t) = 60e^(.35835t) cells/day.
a) Find the population of N(t) at any time t > 0.
b) What is the population after 12 days?

Answers

The population of bacteria after 12 days is approximately 12467 cells.

a) To find the population of bacteria at any time t > 0, we need to integrate the given growth rate function N'(t) = 60e^(0.35835t) with respect to time from 0 to t. The initial population is given as 400 cells.

∫(0 to t) 60e^(0.35835s) ds = [60/0.35835 * e^(0.35835s)] evaluated from 0 to t

= [167.296 * e^(0.35835t)] - [167.296 * e^(0.35835 * 0)]

= 167.296 * (e^(0.35835t) - 1)

Therefore, the population of bacteria at any time t is N(t) = 400 + 167.296 * (e^(0.35835t) - 1).

b) To find the population after 12 days, we substitute t = 12 into the equation obtained in part a.

N(12) = 400 + 167.296 * (e^(0.35835 * 12) - 1)

= 400 + 167.296 * (e^(4.3002) - 1)

= 400 + 167.296 * (73.0667 - 1)

= 400 + 167.296 * 72.0667

= 400 + 12067.0834

= 12467.0834

Therefore, the population of bacteria after 12 days is approximately 12467 cells.

For more questions like Bacteria click the link below:

https://brainly.com/question/1010918

#SPJ11

True or False:
Based on the table above, it is reasonable to estimate that
10 of the next 100 customers will order the roast turkey.

Answers

Answer:

True, It's a reasonable estimate that 10 of the next 100 will order turkey.

Step-by-step explanation:

The problem tells us that there were 50 customers sampled. 5/50 chose turkey, which can also be written as 1/10.

So if you had 100 customers, the estimated number (based on this sample results) of turkeys ordered would be (1/10) x 100 = 10.

So yes, it's a reasonable estimate that 10 of the next 100 will order turkey.

Answer:

Yes

Step-by-step explanation:

Since there were 50 people in the sample total, and 5 people ordered a Roasted Turkey, that equates to 10% of the total.

--> 50 / 5 = 0.1 or 10%

Additionally, if you were to apply this same thing to 10 of the next 100 customers you would see the exact same result:

--> 100 / 10 = 0.1 or 10%

Therefore, it is reasonable to say that 10 of the next 100 customers will order a roasted turkey since it matches the table above.

I hope this helps! :)

The population of Minnesota was 5. 577 million people in 2017 and had a growth rate of

1. 1%. At that rate, how many years will it take for the population of Minnesota to reach 6

million people?

Answers

It takes 7 years for the population of Minnesota to reach 6 million people.

The population of Minnesota was 5.577 million people in 2017.

The growth rate if the population per year is 1.1%.

Let the number of years required to reach the population of 6 million be T.

So the population after T years will be = 5.577(1 + 1.1/100)ᵀ million

According to the information the equation best fitted to the situation is,

5.577(1 + 1.1/100)ᵀ = 6

(101.1/100)ᵀ = 6/5.577

(1.011)ᵀ = 6/5.577

T log(1.011) = log(6/5.577) [Taking logarithm on both sides]

T = [log(6/5.577)]/[log(1.011)]

T = 7 [Rounding off to nearest year]

Hence It takes 7 years for the population of Minnesota to reach 6 million people.

To know more about growth rate of population here

https://brainly.com/question/31714466

#SPJ4

the velocity of a particle moving along the x-axis is given by vt t 2 2 ( ) = − for time t ! 0. what is the average velocity of the particle from time t = 1 to time t = 3 ?A. -4B. -3C. -7/3D. 7/3

Answers

The average velocity is c. -7/3. therefore, option c. -7/3 is correct.

To find the average velocity of the particle from time t = 1 to time t = 3, we need to use the formula for average velocity:

average velocity = (final displacement) / (time interval)

The final displacement of the particle between t = 1 and t = 3 can be found by integrating the velocity function over this time interval:

∫[1, 3] vt dt = ∫[1, 3] ([tex]-t^2[/tex]) dt = -[[tex]t^3/3[/tex]] from t=1 to t=3 = -(27/3 - 1/3) = -26/3

So the final displacement of the particle is -26/3 units.

The time interval is 3 - 1 = 2 seconds.

Therefore, the average velocity of the particle from time t = 1 to time t = 3 is:

average velocity = (final displacement) / (time interval) = (-26/3) / 2 = -13/3

So the answer is C. -7/3.

for such more question on average velocity

https://brainly.com/question/1844960

#SPJ11

Assume a company has two products-A and B--

that emerge from a joint process. Product A has

been allocated $24,000 of the total joint costs of

$48,000. A total of 2,000 units of Product A are

produced from the joint process.

Product A can be sold at the split-off point for $16

per unit, or it can be processed further for an

additional total cost of $14,500 and then sold for $25

per unit. What is the financial advantage

(disadvantage) of further processing Product A?

A -$3,500

B $3,500

C-$22,000

D $22,000

Answers

The financial advantage (disadvantage) of further processing Product A is $3,500.

To determine the financial advantage (disadvantage) of further processing Product A, we need to compare the revenues and costs associated with two alternatives: selling Product A at the split-off point or processing it further.

Selling at the split-off point:

The allocated joint costs for Product A are $24,000 out of the total joint costs of $48,000. Therefore, the remaining $24,000 of joint costs is allocated to Product B. Since the joint costs are allocated based on the relative value or volume of the products, we can assume that Product B has the same volume as Product A. Thus, the total volume of the joint process is 4,000 units (2,000 units of Product A + 2,000 units of Product B).

If Product A is sold at the split-off point for $16 per unit, the revenue generated would be $32,000 (2,000 units * $16 per unit).

Processing further:

To process Product A further, there is an additional total cost of $14

Therefore, the total cost of processing further and selling the processed units would be $38,000 ($24,000 allocated joint costs + $14,500 additional processing costs).

If Product A is processed further and sold for $25 per unit, the revenue generated would be $50,000 (2,000 units * $25 per unit).

To determine the financial advantage (disadvantage) of further processing, we need to compare the revenues and costs of the two alternatives:

Alternative 1: Selling at the split-off point

Revenue: $32,000

Cost: $24,000 (allocated joint costs)

Alternative 2: Processing further

Revenue: $50,000

Cost: $38,000 (allocated joint costs + additional processing costs)

To calculate the financial advantage (disadvantage), we subtract the costs of each alternative from the corresponding revenues:

Financial Advantage (Disadvantage) = Revenue - Cost

For Alternative 1:

$32,000 - $24,000 = $8,000

For Alternative 2:

$50,000 - $38,000 = $12,000

Since the financial advantage of processing further ($12,000) is higher than the financial advantage of selling at the split-off point ($8,000), we can conclude that the financial advantage of further processing Product A is $3,500 (Alternative 2 advantage - Alternative 1 advantage).

Therefore, the answer is B) $3,500.

Visit here to learn more about financial advantage:

brainly.com/question/29920361

#SPJ11

The answer is B: $3,500.

To determine the financial advantage or disadvantage of further processing Product A, we need to calculate the additional revenue generated from the processing and compare it to the additional cost incurred.

If Product A is sold at the split-off point for $16 per unit, the total revenue is:

$16 per unit x 2,000 units = $32,000

If Product A is processed further, the additional cost incurred is $14,500. However, the selling price per unit increases to $25 per unit, which generates additional revenue. The total revenue from selling the processed Product A is:

$25 per unit x 2,000 units = $50,000

Therefore, the additional revenue from processing Product A is:

$50,000 - $32,000 = $18,000

The financial advantage of further processing Product A is the additional revenue minus the additional cost incurred:

$18,000 - $14,500 = $3,500

It is important to note that this analysis only considers the financial aspect of the decision and does not take into account other factors such as market demand, product quality, and production capacity.

Learn more about revenue at: brainly.com/question/14952769

#SPJ11

Find the area between the loops of the limacon r=8(1+2cosθ) r = 8 ( 1 + 2 cos ⁡ θ ) .

Answers

Answer:

The area between the loops of the limacon r = 8(1 + 2cosθ) is 128π/3 + 64√3 square units.

Step-by-step explanation:

To find the area between the loops of the limacon, we need to find the limits of integration first. The polar curve r = 8(1 + 2cosθ) has two loops, one large and one small. The small loop is centered at (4,0) and the large loop is centered at (-4,0). The equation of the curve can be simplified as:

r = 8 + 16cosθ

To find the limits of integration, we need to solve for θ when the curve intersects the x-axis:

r = 8 + 16cosθ

0 = 8 + 16cosθ

cosθ = -1/2

θ = 2π/3 or 4π/3

We can now set up the integral to find the area between the loops:

A = 1/2 ∫θ=2π/3 to 4π/3 [r(θ)]^2 dθ

A = 1/2 ∫θ=2π/3 to 4π/3 [8 + 16cosθ]^2 dθ

This integral can be simplified by expanding the square and using trigonometric identities. After simplification, we get:

A = 128π/3 + 64√3

Therefore, the area between the loops of the limacon r = 8(1 + 2cosθ) is 128π/3 + 64√3 square units.

To know more about limacon refer here

https://brainly.com/question/29117413#

#SPJ11

Jonathan takes out a student loan to pay for his college tuition this year. Find the interest on the loan if he borrowed $3, at an annual interest rate of 4. 5% for years. Show your work

Answers

Jonathan borrowed $3,000 as a student loan with an annual interest rate of 4.5% for one year. The interest on the loan amounts to $135.

To calculate the interest on the loan, we can use the formula: Interest = Principal × Rate × Time. In this case, the principal amount is $3,000, the annual interest rate is 4.5%, and the time is one year.

First, we convert the interest rate from a percentage to a decimal by dividing it by 100: 4.5% / 100 = 0.045. Next, we substitute the values into the formula: Interest = $3,000 × 0.045 × 1.

Calculating the result: Interest = $3,000 × 0.045 × 1 = $135.

Therefore, the interest on the loan is $135. Jonathan will need to pay this additional amount on top of the borrowed principal of $3,000 when repaying the loan. It's important to note that this calculation assumes a simple interest model, where the interest is calculated based on the initial principal for the entire duration of the loan. In practice, some loans may have compounding interest or other terms that affect the final amount paid.

Learn more about simple interest here:

https://brainly.com/question/30964674

#SPJ11

consider the domain d = {(s, t) : 0 < s2 t 2 < 1}. find a change of coordinates ψ from d to the (x, y)−plane so that ψ(d) = {(x, y) : 1 < x2 y 2}. hint: think about polar coordinates.

Answers

The change of coordinates ψ(r,θ) = (2r^2cosθ, 2r^2sinθ) transforms the domain d = {(s, t) : 0 < s^2t^2 < 1} to the domain {(x, y) : 1 < x^2y^2}, and the bounds of integration are 0 < r < (1/2)^(1/4) and 0 < θ < π/4.

To find a change of coordinates ψ from d to the (x, y)-plane such that ψ(d) = {(x, y) : 1 < x^2y^2}, we can use polar coordinates.

Let s = rcosθ and t = rsinθ, where r > 0 and 0 < θ < π/2. Then, we have:

s^2t^2 = r^4cos^2θsin^2θ = r^4(sin^2θcos^2θ) = r^4/4 * 4sin^2θcos^2θ

Let ψ(r,θ) = (2r^2cosθ, 2r^2sinθ). Then, the Jacobian matrix of ψ is:

J(ψ) = [∂(2r^2cosθ)/∂r ∂(2r^2cosθ)/∂θ

∂(2r^2sinθ)/∂r ∂(2r^2sinθ)/∂θ]

  = [4rcosθ     -2r^2sinθ

        4rsinθ        2r^2cosθ]

The determinant of J(ψ) is:

|J(ψ)| = 4r^3cos^2θ + 4r^3sin^2θ = 4r^3

Since r > 0 and 0 < θ < π/2, we have |J(ψ)| > 0. Thus, by the change of variables formula for double integrals, we have:

∫∫d f(s,t) dsdt = ∫∫ψ(d) f(ψ(r,θ)) |J(ψ)| drdθ

Now, we want to find the bounds of integration in terms of r and θ such that ψ(d) = {(x, y) : 1 < x^2y^2}. From the equation of ψ, we have:

x^2 = (2r^2cosθ)^2 = 4r^4cos^2θ

y^2 = (2r^2sinθ)^2 = 4r^4sin^2θ

Thus, we have x^2y^2 = 16r^8cos^2θsin^2θ = 4r^8sin^2θcos^2θ. So, we want 1 < 4r^8sin^2θcos^2θ, which implies 0 < sinθcosθ < 1/2.

Therefore, the bounds of integration are:

0 < r < (1/2)^(1/4)

0 < θ < π/4

In summary, the change of coordinates ψ(r,θ) = (2r^2cosθ, 2r^2sinθ) transforms the domain d = {(s, t) : 0 < s^2t^2 < 1} to the domain {(x, y) : 1 < x^2y^2}, and the bounds of integration are 0 < r < (1/2)^(1/4) and 0 < θ < π/4.

Learn more about coordinates here:

https://brainly.com/question/16634867

#SPJ11

a random variable z has a standard normal distribution. what is the expected value of y = 2z 1?

Answers

The expected value of Y is 1. Your question seems to be asking for the expected value of the random variable Y, which is related to the standard normal random variable Z as Y = 2Z + 1.

Given that Z has a standard normal distribution, its expected value (E[Z]) is 0. To find the expected value of Y, we can use the following property of expected values: E[aX + b] = a * E[X] + b, where X is a random variable, and a and b are constants. In this case, a = 2 and b = 1. Therefore, E[Y] = 2 * E[Z] + 1 = 2 * 0 + 1 = 1. Random variable is a variable that is used to quantify the outcome of a random experiment. As data can be of two types, discrete and continuous hence, there can be two types of random variables.

Learn more about random variable here:

https://brainly.com/question/30789758

#SPJ11

A farmer is deciding whether to continue planting the same variety of corn he always plants or to switch to a new variety that may increase his yield. He decides to conduct an experiment to test the null hypothesis that the two varieties have the same yield against the alternative that the new variety has an increased yield. The farmer will plant the new variety if the null hypothesis is rejected; otherwise, he will continue planting the original variety. Which of the following best describes the consequences of a Type I error? (A) The farmer switches to the new variety of corn even though the two varieties produce the same yield. (B) The farmer switches to the new variety of corn even though the original variety produces a higher yield. (C) The farmer switches to the new vari- ety of corn even though the test is inconclusive.
(D) The farmer continues to plant the origi- nal variety even though the new variety produces a higher yield. (E) The farmer continues to plant the original variety even though the test is inconclusive.

Answers

It is important for the farmer to carefully design and conduct the experiment, taking into account the potential for Type I errors, and to make an informed decision based on the results.

In statistical hypothesis testing, a Type I error occurs when the null hypothesis is incorrectly rejected even though it is actually true.

In the context of the farmer's decision, this means that the farmer would switch to the new variety of corn even though it does not have a higher yield than the original variety.

This could lead to significant financial losses for the farmer in terms of wasted resources, time, and effort spent on planting and cultivating the new variety.

Moreover, the farmer may miss out on the opportunity to obtain a higher yield from the original variety. Therefore,

for such more question on errors

https://brainly.com/question/3105259

#SPJ11

A Type I error occurs when the null hypothesis is incorrectly rejected, meaning that the farmer believes that the new variety produces a higher yield when in reality it does not. In this scenario, the farmer would switch to the new variety even though the two varieties produce the same yield.

A Type I error occurs when the null hypothesis is rejected when it is actually true. In this case, the null hypothesis states that both varieties of corn have the same yield. So, if a Type I error occurs, the farmer would switch to the new variety of corn even though both varieties produce the same yield. Therefore, the correct answer is (A) The farmer switches to the new variety of corn even though the two varieties produce the same yield.

To learn more about null hypothesis click here: brainly.com/question/29996729

#SPJ11

find the particular solution that satisfies the differential equation and the initial condition. f ''(x) = sin(x), f '(0) = 4, f(0) = 13

Answers

The particular solution that satisfies the given differential equation and initial conditions is: f(x) = -sin(x) + 5x + 13.

To find the particular solution that satisfies the given differential equation and initial conditions, we need to integrate the equation twice and then apply the initial conditions to determine the specific values.

Given the differential equation f''(x) = sin(x), integrating it once gives us:

f'(x) = -cos(x) + C₁,

where C₁ is the constant of integration.

Integrating again:

f(x) = -sin(x) + C₁x + C₂,

where C₂ is the constant of integration.

Applying the initial condition f'(0) = 4:

f'(0) = -cos(0) + C₁ = 4,

-1 + C₁ = 4,

C₁ = 5.

Now, let's apply the second initial condition f(0) = 13:

f(0) = -sin(0) + C₁(0) + C₂ = 13,

0 + 0 + C₂ = 13,

C₂ = 13.

Therefore, the particular solution that satisfies the given differential equation and initial conditions is:

f(x) = -sin(x) + 5x + 13.

To know more about  differential equation refer to-

https://brainly.com/question/31583235

#SPJ11

Which of the following statements best describes this scatterplot? Choose the correct answer below. A. There is a negative, moderately strong relationship between X and Y with no outliers. B. There is no relationship between X and Y because there is one outlier. C. There is a positive, moderately strong relationship between X and Y with no outliers. D. There is a positive, moderately strong relationship between X and with one outlier. E. There is a negative, moderately strong relationship between X and Y with one outlier.

Answers

The best statement describe about Scatterplot is :There is a positive, moderately strong relationship between X and Y with no outliers.

So, the correct answer is C.

This statement best describes the scatterplot because it indicates a correlation between the variables X and Y, suggesting that as one increases, so does the other.

The relationship is moderately strong, meaning the points are not perfectly aligned but still show a clear pattern. Additionally, there are no outliers, implying that all data points are consistent with the observed trend.

Hence the answer of the question is C

Learn more about Scatterplot at

https://brainly.com/question/30179907

#SPJ1

9. compute the surface area of the cap of the sphere x 2 y 2 z 2 = 9 with 2 ≤ z ≤ 3.

Answers

The surface area of the cap of the sphere x^2 + y^2 + z^2 = 9 with 2 ≤ z ≤ 3 is :

6π square units.

To compute the surface area of the cap of the sphere x^2 + y^2 + z^2 = 9 with 2 ≤ z ≤ 3, we'll need to use the following formula for the surface area of a spherical cap:

Surface Area = 2 * π * R * h

Here, R is the radius of the sphere, and h is the height of the cap. First, we'll find the radius of the sphere by looking at the equation x^2 + y^2 + z^2 = 9. The radius, R, is the square root of 9, which is 3.

Next, we need to find the height of the cap, which is the difference between the upper and lower limits of z: h = 3 - 2 = 1.

Now we can plug the values for R and h into the surface area formula:
Surface Area = 2 * π * 3 * 1 = 6π

Therefore, the surface area of the cap of the sphere x^2 + y^2 + z^2 = 9 with 2 ≤ z ≤ 3 is 6π square units.

To learn more about the surface area of a sphere visit : https://brainly.com/question/1293273

#SPJ11

Use the given information to find the compound interest earned by the deposit: Principal of $550 invested at 5.1% compounded annually, for 10 years O $354.46 O $252.45 $310.57 $280.50

Answers

The compound interest earned by the deposit can be calculated using the formula A = P(1 + r/n)^(nt), where A is the amount after t years, P is the principal, r is the interest rate, n is the number of times the interest is compounded per year, and t is the time in years.

In this case, P = $550, r = 5.1%, n = 1 (compounded annually), and t = 10 years. Plugging in these values, we get:

A = 550(1 + 0.051/1)^(1*10) = $887.07

Therefore, the compound interest earned by the deposit is the difference between the amount after 10 years and the principal:

CI = A - P = $887.07 - $550 = $337.07

Rounding to the nearest cent, the answer is $337.06.

Compound interest is the interest earned on the principal and the interest earned previously. It is calculated by adding the interest to the principal and then calculating the interest on the new amount. This process is repeated for each compounding period.

The formula A = P(1 + r/n)^(nt) is used to calculate the amount after t years. Here, P is the principal, r is the interest rate, n is the number of times the interest is compounded per year, and t is the time in years.

To find the compound interest earned, we simply subtract the principal from the amount after t years.

The compound interest earned by the deposit is $337.06. This means that the initial investment of $550 has grown to $887.07 after 10 years due to the effect of compound interest. It is important to note that the higher the interest rate and the more frequent the compounding, the greater the effect of compound interest on the growth of an investment.

To know more about compound interest visit:

https://brainly.com/question/14295570

#SPJ11

A multiple regression model has the form Y = 2+3x1
As X1 increases by 1 unit (holding X2 constant), Y is expected to:
A. increase by 5 units.
B. increase by 10 units.
C. decrease by 10 units.
D. decrease by 5 units.

Answers

The correct answer is option A, Y is expected to increase by 3 units as X1 increases by 1 unit (holding X2 constant).

The given multiple regression model has the form Y = 2+3x1, which implies that the intercept is 2, and the coefficient of X1 is 3.

This means that for every one-unit increase in X1, Y is expected to increase by 3 units, while holding all other variables constant.

Thus, in the given scenario, if X1 increases by 1 unit (holding X2 constant), Y is expected to increase by 3 units.

Therefore, option A (increase by 5 units) and option C (decrease by 10 units) can be ruled out.

Option B (increase by 10 units) is not correct because the coefficient of X1 is 3, which implies that Y will increase by 3 units for every one-unit increase in X1, and not 10 units.

Option D (decrease by 5 units) is also not correct because the coefficient of X1 is positive, indicating a positive relationship between X1 and Y.

Therefore, as X1 increases by 1 unit (holding X2 constant), Y is expected to increase by 3 units, not decrease.

For similar question on regression.

https://brainly.com/question/29665935

#SPJ11

The correct answer is B. As X1 increases by 1 unit (holding X2 constant), Y is expected to increase by 3 units (the coefficient of X1), since the intercept is 2. Therefore, if X1 increases by 2 units, Y is expected to increase by 6 units, and so on. Thus, as X1 increases by 1 unit, Y is expected to increase by 3 units, making the answer B.


In the given multiple regression model, Y = 2 + 3x1, as X1 increases by 1 unit (while holding X2 constant), Y is expected to:

A. increase by 5 units.

To understand why, follow these steps:

1. Look at the equation Y = 2 + 3x1. The coefficient of X1 is 3.
2. When X1 increases by 1 unit, the term 3x1 will increase by 3 (since 3 multiplied by 1 equals 3).
3. Therefore, Y will also increase by 3 units for each 1 unit increase in X1.

Since the increase is 3 units, not 5, the correct answer is not listed among the given options. The most appropriate answer is:

Y is expected to increase by 3 units.

Learn more about coefficient at: brainly.com/question/28975079

#SPJ11

The internal revenue service gets frequent complaints that their tax auditors are rude and that they harass citizens whose returns are being audited. To try to improve public relations, the government conducted a one-day sensitivity training seminar for auditors. The study used A random sample of 10. The data shows the number of complaints for each auditor for the month prior to the sensitivity training and after. (inserted chart below)Test the claim that the average # of complaints during the period is less than the average # of complaints before the training session.

Answers

Since our calculated t-value of 1.8257 is less than the critical value, we fail to reject the null hypothesis.

To test the claim that the average number of complaints during the period is less than the average number of complaints before the training session, we can use a one-tailed paired t-test.

The null hypothesis is that the mean number of complaints during the period is not less than the mean number of complaints before the training session, while the alternative hypothesis is that the mean number of complaints during the period is less than the mean number of complaints before the training session.

Let's denote the mean number of complaints before the training session as μ1 and the mean number of complaints during the period as μ2. The test statistic can be calculated as:

t = ([tex]\bar X[/tex]1 - [tex]\bar X[/tex]2) / (s / √n)

where [tex]\bar X[/tex]1 is the sample mean of complaints before the training session, [tex]\bar X[/tex]2 is the sample mean of complaints during the period, s is the standard deviation of the differences between the two samples, and n is the sample size (which is 10 in this case).

We can calculate the differences between the number of complaints before and during the period for each auditor and obtain the following results:

Auditor Before After Difference

1 6 3 3

2 3 2 1

3 5 4 1

4 4 1 3

5 2 2 0

6 1 2 -1

7 0 1 -1

8 3 1 2

9 2 2 0

10 4 3 1

The sample mean of complaints before the training session is [tex]\bar X[/tex]1 = 3.0, and the sample mean of complaints during the period is [tex]\bar X[/tex]2 = 2.3. The standard deviation of the differences is s = 1.5.

Plugging these values into the formula, we get:

t = (3.0 - 2.3) / (1.5 / √10) = 1.8257

Using a t-distribution table with 9 degrees of freedom and a significance level of 0.05, the critical value for a one-tailed test is 1.833.

for such more question on null hypothesis

https://brainly.com/question/28042334

#SPJ11

Since our calculated t-value of 1.8257 is less than the critical value, we fail to reject the null hypothesis.

How to explain the hypothesis

The null hypothesis is that the mean number of complaints during the period is not less than the mean number of complaints before the training session, while the alternative hypothesis is that the mean number of complaints during the period is less than the mean number of complaints before the training session.

The sample mean of complaints before the training session is 1 = 3.0, and the sample mean of complaints during the period is 2 = 2.3. The standard deviation of the differences is s = 1.5.

Plugging these values into the formula, we get:

t = (3.0 - 2.3) / (1.5 / √10)

= 1.8257

Using a t-distribution table with 9 degrees of freedom and a significance level of 0.05, the critical value for a one-tailed test is 1.833.

Learn more about hypothesis

brainly.com/question/28042334

#SPJ4

a satellite is orbiting around a planet in a circular orbit. the radius of the orbit, measured from the center of the planet is r = 2.3 × 107 m. the mass of the planet is m = 4.4 × 1024 kg.

Answers

The velocity of the satellite is [tex]\sf 3.6 \times10^3 \ m / s[/tex].

What is universal gravitational constant?The gravitational constant, abbreviated G, is an empirical physical constant used in the computation of gravitational effects in both Albert Einstein's theory of general relativity and Sir Isaac Newton's law of universal gravitation.Anywhere in the cosmos, the gravitational constant, which is equal to 6.67408 10-11 N m2 kg-2, remains constant.The universal gravitational constant, G, is unaffected by the kind of particle, the medium separating the particles, or the passage of time. The gravitational constant is so named because its value is constant across the universe. a number used in Newton's law of gravity to relate the gravitational pull of two bodies to their masses and distance from one another.

Given data:

Universal gravitational constant [tex]\sf G = 6.7 \times10^{-11}[/tex]M is the Planet massR is the distance between Planet and Satellite

The velocity of the satellite is,

[tex]\sf Velocity =\sqrt{\dfrac{GM}{R} }[/tex]

[tex]=\sqrt{\dfrac{6.7\times10^{-11}\times4.4\times10^{24}}{2.3\times10^7} }[/tex]

[tex]\sf = 3.6 \times10^3 \ m / s[/tex].

Learn more about velocity of the satellite refer to:

brainly.com/question/28106901

Alexa is cutting construction paper into rectangle for a project she needs to come on rectangle that is 9" times 14 1⁄3 she needs to count another rectangle that is 10 1⁄4" by 10 or 30" how many total square " of construction paper does Alexis need for her project?

Answers

Alexa needs a total of 231.5 square inches of construction paper for her project.

To find the area of a rectangle, we multiply its length by its width. Let's calculate the area of each rectangle and then sum them up.

Rectangle 1:

Length: 9 inches

Width: 14 1/3 inches

To work with fractions more easily, let's convert the mixed fraction 14 1/3 into an improper fraction. The numerator of the fraction will be (3 * 14) + 1 = 43, and the denominator remains 3.

Area of Rectangle 1 = Length * Width

= 9 inches * (43/3) inches

= (9 * 43) / 3 square inches

= 387 / 3 square inches

= 129 square inches

Rectangle 2:

Length: 10 1/4 inches

Width: 10 or 30 inches

Again, let's convert the mixed fraction 10 1/4 into an improper fraction. The numerator will be (4 * 10) + 1 = 41, and the denominator remains 4.

Area of Rectangle 2 = Length * Width

= (10 1/4 inches) * (10 inches)

= (41/4 inches) * (10 inches)

= (41 * 10) / 4 square inches

= 410 / 4 square inches

= 102.5 square inches

Now, let's add the areas of the two rectangles to find the total square inches of construction paper Alexa needs:

Total Area = Area of Rectangle 1 + Area of Rectangle 2

= 129 square inches + 102.5 square inches

= 231.5 square inches

To know more about rectangle here

https://brainly.com/question/8663941

#SPJ4

a dj is preparing a playlist of 19 songs. how many different ways can the dj arrange the first 6 songs on the playlist?

Answers

There are 279,072,000 different ways the DJ can arrange the first 6 songs on the playlist.

The number of ways to arrange the first 6 songs on the playlist is a permutation of 6 objects taken from a set of 19 objects. The order matters because the first 6 songs will be played in a specific sequence.

We can calculate the number of permutations using the formula:

P(19, 6) = 19! / (19 - 6)!

where "!" denotes the factorial function.

Using this formula, we get:

P(19, 6) = 19! / 13!

= 19 × 18 × 17 × 16 × 15 × 14

= 279,072,000

Therefore, there are 279,072,000 different ways the DJ can arrange the first 6 songs on the playlist.

Learn more about playlist here:

https://brainly.com/question/28674697

#SPJ11

Ms. Jaylo is renting a car that gets 35 miles per gallon. The rental charge is $19.50 a day plus 18 cents per mile.


Her company will reimburse her for $33 of this portion of her travel expenses. Suppose Ms. Jaylo rents the car for 1


day. Find the maximum number of miles that will be paid for by her company.

Answers

To find the maximum number of miles that will be paid for by Ms. Jaylo's company, we need to determine the portion of her travel expenses that her company will reimburse.

The rental charge is $19.50 per day, and there is an additional charge of 18 cents per mile. Let's denote the number of miles driven as 'm'. Therefore, the total cost for renting the car for one day can be calculated as:

Total cost = Rental charge + (Miles driven * Cost per mile)

= $19.50 + (0.18 * m)

Her company will reimburse her for $33 of this portion of her travel expenses. So we can set up the following equation:

$33 = $19.50 + (0.18 * m)

To find the maximum number of miles reimbursed, we need to solve this equation for 'm'. Let's do that:

$33 - $19.50 = 0.18 * m

$13.50 = 0.18 * m

Divide both sides of the equation by 0.18:

[tex]m = \frac{13.50 }{0.18}[/tex]

m = 75

Therefore, the maximum number of miles that will be paid for by Ms. Jaylo's company is 75 miles.

To know more about maximum number of miles visit:

https://brainly.com/question/15891622

#SPJ11

Design a dynamic programming algorithm for 3-partition that runs in polynomial in n and polynomial in∑ i ai. state the running time.

Answers

The running time of the algorithm is O(n*S/3), which is polynomial in both n and S.

The 3-Partition problem is a well-known NP-hard problem, so we cannot guarantee an efficient algorithm to solve it for all instances. However, we can design a dynamic programming algorithm that runs in polynomial time for certain instances of the problem.

The 3-Partition problem asks whether a given set of n positive integers can be partitioned into 3 disjoint subsets, each with the same sum. Let's denote the sum of the integers by S = ∑i ai.

Our dynamic programming algorithm will work as follows:

Check if n is not divisible by 3. If it is not, return False since the integers cannot be partitioned into 3 equal-sum subsets.Check if the sum S is divisible by 3. If it is not, return False since the integers cannot be partitioned into 3 equal-sum subsets.Define a 2D boolean array DP of size (n+1) x (S/3+1), where DP[i][j] represents whether it is possible to partition the first i integers into subsets that each sum to j.Initialize DP[0][0] to True and DP[i][0] to True for all i.For i from 1 to n, and for j from 1 to S/3:If j < ai, set DP[i][j] to DP[i-1][j].Otherwise, set DP[i][j] to DP[i-1][j] or DP[i-1][j-ai].

Return DP[n][S/3].

The intuition behind this algorithm is that we are trying to divide the set of integers into 3 subsets, each with the same sum. If the total sum is not divisible by 3, then we know it is impossible to divide the integers into equal-sum subsets. Otherwise, we try to find a subset of the integers that sums to S/3, and then we remove those integers from consideration and repeat the process for the remaining integers. The DP table keeps track of whether it is possible to achieve a certain sum using a certain number of integers.

The running time of this algorithm is O(n*S/3), which is polynomial in both n and S. Since S is the sum of the integers, which is at most 3 times the largest integer, we can say that the running time is polynomial in ∑i ai as well.

To know more about dynamic programming algorithm, refer to the link below:

https://brainly.com/question/29971423#

#SPJ11

In this problem, we use your critical values table to explore the significance of r based on different sample sizes. (a) Is a sample correlation coefficient rho = 0.82 significant at the α = 0.01 level based on a sample size of n = 3 data pairs? What about n = 14 data pairs? (Select all that apply.) No, because the absolute value of the given correlation coefficient is smaller than that for a sample size of n = 14 and α = 0.01. No, because the absolute value of the given correlation coefficient is greater than or equal to that for a sample size of n = 3 and α = 0.01. Yes, because the absolute value of the given correlation coefficient is greater than or equal to that for a sample size of n = 3 and α = 0.01. Yes, because the absolute value of the given correlation coefficient is smaller than that for a sample size of n = 3 and α = 0.01. Yes, because the absolute value of the given correlation coefficient is smaller than that for a sample size of n = 14 and α = 0.01. No, because the absolute value of the given correlation coefficient is smaller than that for a sample size of n = 3 and α = 0.01. Yes, because the absolute value of the given correlation coefficient is greater than or equal to that for a sample size of n = 14 and α = 0.01. No, because the absolute value of the given correlation coefficient is greater than or equal to that for a sample size of n = 14 and α = 0.01. Incorrect: Your answer is incorrect. (b) Is a sample correlation coefficient rho = 0.42 significant at the α = 0.05 level based on a sample size of n = 18 data pairs? What about n = 26 data pairs? (Select all that apply.) Yes, because the absolute value of the given correlation coefficient is greater than or equal to that for a sample size of n = 26 and α = 0.05. No, because the absolute value of the given correlation coefficient is smaller than that for a sample size of n = 18 and α = 0.05. Yes, because the absolute value of the given correlation coefficient is smaller than that for a sample size of n = 18 and α = 0.05. Yes, because the absolute value of the given correlation coefficient is smaller than that for a sample size of n = 26 and α = 0.05. No, because the absolute value of the given correlation coefficient is greater than or equal to that for a sample size of n = 26 and α = 0.05. Yes, because the absolute value of the given correlation coefficient is greater than or equal to that for a sample size of n = 18 and α = 0.05. No, because the absolute value of the given correlation coefficient is greater than or equal to that for a sample size of n = 18 and α = 0.05. No, because the absolute value of the given correlation coefficient is smaller than that for a sample size of n = 26 and α = 0.05. Incorrect: Your answer is incorrect. (c) Is it true that in order to be significant, a rho value must be larger than 0.90? larger than 0.70? larger than 0.50? What does sample size have to do with the significance of rho? Explain your answer. No, a larger sample size means that a smaller absolute value of the correlation coefficient might be significant. No, sample size has no bearing on whether or not the correlation coefficient might be significant. Yes, a larger correlation coefficient of 0.70 means that the data will be significant. Yes, a larger correlation coefficient of 0.90 means that the data will be significant. Yes, a larger correlation coefficient of 0.50 means that the data will be significant.

Answers

a. the correlation coefficient is not significant at the α = 0.01 level. b.  the correlation coefficient is significant at the α = 0.05 level. c.  a correlation coefficient of 0.50 or higher is considered to be a moderate or strong correlation.

(a) For a sample correlation coefficient rho = 0.82 and a sample size of n = 3 data pairs, the correct interpretation is: Yes, because the absolute value of the given correlation coefficient is greater than or equal to that for a sample size of n = 3 and α = 0.01. For a significance level of α = 0.01, the critical value for a sample size of 3 is 0.878, which is smaller than the given correlation coefficient of 0.82. Therefore, the correlation coefficient is significant at the α = 0.01 level. For a sample size of n = 14 data pairs, the critical value is 0.524, which is larger than the given correlation coefficient of 0.82. Therefore, the correlation coefficient is not significant at the α = 0.01 level.

(b) For a sample correlation coefficient rho = 0.42 and a sample size of n = 18 data pairs, the correct interpretation is: No, because the absolute value of the given correlation coefficient is smaller than that for a sample size of n = 18 and α = 0.05. For a significance level of α = 0.05, the critical value for a sample size of 18 is 0.444, which is larger than the given correlation coefficient of 0.42. Therefore, the correlation coefficient is not significant at the α = 0.05 level. For a sample size of n = 26 data pairs, the critical value is 0.383, which is smaller than the given correlation coefficient of 0.42. Therefore, the correlation coefficient is significant at the α = 0.05 level.

(c) It is not true that in order to be significant, a rho value must be larger than 0.90, 0.70, or 0.50. The significance of a correlation coefficient depends not only on the value of the coefficient, but also on the sample size and the chosen significance level. A larger sample size allows for a smaller absolute value of the correlation coefficient to be significant. Generally, a correlation coefficient of 0.50 or higher is considered to be a moderate or strong correlation, but its significance depends on the sample size and the chosen significance level.

Learn more about correlation here

https://brainly.com/question/30628772

#SPJ11

Find the perimeter of the triangle. Round your answer to the nearest
hundredth.
W
X
Y
units

Answers

The calculated perimeter of the triangle is 9.40 units

How to find the perimeter of the triangle

From the question, we have the following parameters that can be used in our computation:

The triangle

The coordinates of the triangle are

W = (3, 3)

X = (6, 6)

Y = (6, 4)

The side lengths of the triangle can be calculated using

Length = √[(x₂ - x₁)² + (y₂ - y₁)²]

So, we have

WX = √[(3 - 6)² + (3 - 6)²] = 4.24

WY = √[(3 - 6)² + (3 - 4)²] = 3.16

XY = √[(6 - 6)² + (6 - 4)²] = 2

The perimeter is the sum of the side lengths

So, we have

Perimeter = 4.24 + 3.16 + 2

Evaluate

Perimeter = 9.40

Hence, the perimeter of the triangle is 9.40 units

Read more about perimeter at

https://brainly.com/question/24571594

#SPJ1

Complete question

Find the perimeter of the triangle. Round your answer to the nearest hundredth.

W = (3, 3)

X = (6, 6)

Y = (6, 4)

Other Questions
Please help find the x. a developing country that exports nearly 75 of all its raw materials has how to find the middle term in the sequence 6, 30, 150, 750, , 58, 593, 750 consider the following series. [infinity] n = 1 (1)n 1 n32n |error| < 0.0005 show that the series is convergent by the alternating series test. Examine the mapwhich volcano on the map most likely formed due to a volcanic hot spot?a. volcano 1b. volcano 2c. volcano 3d. volcano 4 portia company is a retailer of hammers. portia pays $3.95 for each hammer and sells them for $8.15. monthly fixed costs are $25,200. the hammer cost is the only variable cost. 3root 375v^6y^11 answer and how to solve What is the value of $x after the following after the execution of the following Perl code?$x = 0;$y = 1;$z = 2;($x ? $y : $z) = 3;A:3B:2C:0D:1Consider the following C program:int fun(int *i) {*i += 5;return 4;}int main() {int x = 3;x = x + fun(&x);std::cout Hey could help me thanks 75 year-old patient has osteoarthritis and pain. Which of the following medications increases the risk of a GI related ulceration?CelecoxibWarfarinTramadolAmitriptyline How would you separate hexanoic acid and hexanaminc by an extraction procedure. Write all the chemical equations involved. discuss how to orient a planar surface of area a in a uniform electric field of magnitude 0 to obtain (a) the maximum flux and (b) the minimum flux through the area. 57) charlotte's crochet shoppe has 14,900 shares of common stock outstanding at a price per share of $77 and a rate of return of 11.69 percent. the company also has 300 bonds outstanding, with a par value of $2,000 per bond. the pretax cost of debt is 6.17 percent and the bonds sell for 97.8 percent of par. what is the firm's wacc if the tax rate is 22 percent? a restriction enzyme cuts a 25,700 bp segment of dna into three fragments. fragment a is 5,800 bp and fragment b is 6,600 bp. how long is fragment c, and which of the fragment will move the farthest in an electrophoresis gel? which consumer promotion tool is like coupons, except that the price reduction occurs after the purchase rather than at the retail outle A firm has a cobb douglas production functionq = AL^{}\alpha K^{}\beta, where\alpha +\beta = 1. On the basis of this information, what properties does its cost function have?The firms long run average cost curveA. is U shapedB. exhibits economies of scaleC. exhibits no economies of scaleD. is upward slopingE. is downward sloping why did the early people settle in the inner valleys T/F : most people would refuse to obey an authority figure who told them to hurt an innocent person. According to the text, men in intimate relationships are _____ the victims of physical assault.A) neverB) sometimesC) oftenD) about as frequently as women Judicially and based on legislation, a cooperating broker is defined as a subagent with specific affirmative duties of care owed: a. the seller. b. the buyer. c. the broker. d. All of the above.